16
$\begingroup$

While playing chess, I had a momentary dizzy spell which erased most of my short-term memory. I knew only the current position, and that I was playing White, and that all play so far had been legal, and that it was my turn.

This information was enough to deduce that there existed a legal move which would checkmate my opponent, but not enough to deduce what the move actually was.

What might the position have been?

(I knew which side of the board was White and which was Black, so there was no ambiguity about which way pawns could move. More generally, this is not intended as a lateral thinking puzzle.)

$\endgroup$
7
  • 1
    $\begingroup$ I believe the solution is not unique and is related to en passant or castling. $\endgroup$ Commented yesterday
  • 1
    $\begingroup$ Yes - there isn't a unique solution. $\endgroup$ Commented yesterday
  • 1
    $\begingroup$ @GentlePurpleRain White can tell that there exists a mate in one. $\endgroup$ Commented 22 hours ago
  • 1
    $\begingroup$ @Cheshire_the_Maomao it's referring to the list of information that White knew given in the first paragraph. $\endgroup$ Commented 10 hours ago
  • 1
    $\begingroup$ Rather than knowing "the current position", it would be more precise to say "the board configuration of pieces" or similar. Note that a "position" carries information about castling rights and e.p. too. $\endgroup$ Commented 7 hours ago

3 Answers 3

7
$\begingroup$

In this position:

position 2R1B1R1/2P1k1P1/2P1P1P1/2PpKpP1/8/8/8/8 w - d6 0 1

White can mate with

cxd6# or gxf6# en passant, depending on whether black just moved d7-d5 to f7-f5. No other move is mate-in-one.

Observe that

Black couldn't have made any other move, since a one-space pawn move would start with White in check, and a King move would start from an impossible double check. This differs from other solutions that rely on the game being over by stalemate if en passant is impossible.
For completeness, we should ensure this position is reachable. Prior to Black's moving d7-d5 or f7-f5, White could have gone Ra8-c8 capturing black's Queen, which Black just moved there. There's not much constraint from here and plenty of slack for White's pawns to make enough captures to get into the needed files, so I hope I can get away with not providing a proof game.

$\endgroup$
1
  • $\begingroup$ Nice, this is very similar to the position I had in mind! $\endgroup$ Commented 10 hours ago
10
$\begingroup$

In the following position:

White pawns on d5, f5, h5. Black pawns on d6, e5, g5, h6. White king on h8. Black king on f7. White knight on g8. Black knight on f6. Black rook on f8. Black bishop on e8.

White could checkmate with

en passant, depending on Black's last move.

We don't know which checkmates are playable, but we know that there's at least one because

otherwise, the position would be stalemate, the game would be over, and it wouldn't be our turn!

$\endgroup$
1
  • 1
    $\begingroup$ I hadn't thought about using stalemate in this way. Very clever! $\endgroup$ Commented 22 hours ago
2
$\begingroup$

How about Example Chess Board

R3B2R/4k3/8/3pPp2/4NN2/8/8/7K w - (d|f)6 0 1

Which we get by the proof game (for en passant to f6):

1. e4 e5
2. Qh5 g5
3. Qxh7 Nh6
4. Qxh8 c6
5. Qxh6 Bg7
6. Qxg7 Qf6
7. Qxf6 b6 
8. Qxc6 Ke7
9. Qxc8 a6
10. Qxa6 Nc6
11. Qxb6 Rh8
12. Qxc6 Rxh2
13. Qg6 Rxg2
14. Qxg5+ Ke8
15. Qe3 Rxf2 
16. Qd3 Rxd2 
17. Qc3 Rxc2 
18. Qc4 Rxb2 
19. Qa4 Rxa2 
20.Qc4 Rc2 
21. Qc5 Rxc1+ 
22. Kf2 Rxc5 
23. Ra5 Rb5 
24. Rxb5 Ke7 
25. Rc5 Ke8 
26. Bb5 Ke7 
27. Nc3 Ke8 
28. Rxe5+ Kf8 
29. Rc5 Ke8 
30. Rc7 Ke7 
31. Ra7 Ke8 
32. Ra8+ Ke7
33. Rhh8 Ke6 
34. Nf3 Ke7 
35. Kg1 Ke6 
36. Kh1 Ke7 
37. Ng5 Kd6 
38. e5+ Ke7 
39. Nge4 Ke6 
40. Ne2 Ke7 
41. N2g3 Ke6
42. Bc6 d5 
43. Be8 Ke7 
44. Rh7 Ke6 
45. Rh6+ Ke7 
46. Rh8 Ke6 
47. Nh5 Ke7 
48. Nf4 f5 
49. exf6#

The proof for the other option is the same through Move 41:

42. Be2 Ke7 43. Bh5 Ke6 44. Ne2 f5 45. Be8 Ke7 46. Nf4 d5 exd6#
$\endgroup$
2
  • 2
    $\begingroup$ Could black have moved a pawn one space last turn so that no en passant is possible? $\endgroup$ Commented 23 hours ago
  • $\begingroup$ @xnor Dammit, I forgot to consider that. $\endgroup$ Commented 23 hours ago

Your Answer

By clicking “Post Your Answer”, you agree to our terms of service and acknowledge you have read our privacy policy.

Start asking to get answers

Find the answer to your question by asking.

Ask question

Explore related questions

See similar questions with these tags.